PT10.S4.Q9 - There is is relatively little room for growth

bililililabilililila Member
edited December 2019 in Logical Reasoning 12 karma

I'm thinking that maybe C is wrong because the mergers does no good to the market share, and the newly merged companies still have to gain market share by purchasing competitors. However, D indicates that since some producers have been squeezed out of the market, there are some space for new companies which can gain their market share through more aggressive marketting. That's why D is the correct answer. i'm not sure if my explaination is correct, please help.

Admin note: edited title; please use the format of "PT#.S#.Q# - [brief description]"

Comments

  • thisisspartathisissparta Alum Member
    edited December 2019 1363 karma

    Re: C, you're right. Not to mention, we don't know if a decline in profits necessarily implies a decline in market share. The argument C purports doesn't "wreck" the argument that the stimulus makes.

    Re: D - I think your reasoning might be a bit off here. D is right because it introduces an alternative explanation on how a company's market share can grow. The stimulus suggests that the only way a company can grow its market share is through mergers (and not aggressive marketing). D, in contrast, states that, in fact, it isn't mergers but rather cost-cutting measures implemented by bigger firms that has caused an increase in their market share.

    Hope this helps! :)

Sign In or Register to comment.